LSAT and Law School Admissions Forum

Get expert LSAT preparation and law school admissions advice from PowerScore Test Preparation.

 Administrator
PowerScore Staff
  • PowerScore Staff
  • Posts: 8929
  • Joined: Feb 02, 2011
|
#80562
Complete Question Explanation

Flaw, CE. The correct answer choice is (E).

The conclusion of the argument is at the end: "if we want there to be even fewer parking violations, the fines should be raised again." But the reasoning to get to this conclusion makes a causal assumption, namely that when the fines were raised that is what caused the parking violations to drop by 50%. Is that a reasonable assumption? It may seem so at first glance, but the problem is that there is another possible cause contained right in the argument: maybe it was the newly opened parking garage, with its many new parking spaces, that lead to fewer violations. If there are now many more available spaces, maybe it is easier to avoid illegally parking and getting a fine. And if this is the case, then raising the fines again wouldn't necessarily lead to fewer violations.

The flaw here is relying on the causal assumption that it was the higher fines that caused the fewer violations, and not realizing there was an alternate cause in the newly added parking garage spaces.

Answer choice (A): This cab be a tricky answer since it throw around the words "cause" and "effect," but the relationship described is not what we have above, which was ignoring a possible alternate cause.

More specifically, the temporal relationship between the fines and the number of parking violations is pretty clearly established in the stimulus, so if we were looking for a flaw that would not be it. For (A) to be the correct answer, the official would have to have said that the drop in parking violations was what caused the fines to be raised, and that is not what the official says.

Answer choice (B): The problem here is the phrase "at least as much." The Official simply said "if we want there to be fewer parking violations," (italics added) and didn't say they had to be "at least as much as" before.

More expansively, the main issue with answer choice (B) is that it does not address the official's actual argument. The official did not argue that raising the fines would results in another 50% (or more) decrease in parking violations, they argued that raising the fines would decrease parking violations by an indefinite amount. So, how can the flaw in the argument be something the argument never concluded in the first place? It can't!

Let's say that six months ago (before any fines were raised), there were about 100 parking violations on average. After the fines are raised and the garage was opened, we now only have 50 parking violations (a reduction of 50%). The Official then says raising fines again will result in even fewer parking violations. This could mean that the Official believes raising fines again will result in now only 49 violations, or 30, or 10 - we aren't sure because all of these are even fewer than 50! In order to assert what (B) says, the Official would have to be arguing that there would now be only 25 parking violations or fewer (which equates to the same 50% reduction seen the first time). However, as previously discussed, we can't really say that's what the Official is arguing.

Answer choice (C): This point is not part of the argument. Ultimately, the answer is not a flaw because while it's true that the author makes no mention of any financial benefits of parking fines, that isn't relevant to their conclusion. The author is not saying that we DO want fewer parking violations. They are saying IF we want fewer violations, we should again raise the fees. Regardless, the financial benefits to the city are never discussed.

Answer choice (D): The argument never talks about or assumes what people want to do, just what happened factually and how higher fines will make them act in the future.

Answer choice (E): This is the correct answer choice. Answer E is saying that the author failed to rule out or consider that alternate cause for the decrease in violations, and "fails to rule out an alternate cause" is a classic causal flaw.
User avatar
 emilyjmyer
  • Posts: 48
  • Joined: May 11, 2022
|
#95583
I see why E is correct, but why is B incorrect?
User avatar
 katehos
PowerScore Staff
  • PowerScore Staff
  • Posts: 184
  • Joined: Mar 31, 2022
|
#95593
Hi Emily,

The main issue with answer choice (B) is that it does not address the official's actual argument. The official did not argue that raising the fines would results in another 50% (or more) decrease in parking violations, they argued that raising the fines would decrease parking violations by an indefinite amount. So, how can the flaw in the argument be something the argument never concluded in the first place? It can't!

I hope this helps! :)
-Kate
User avatar
 emilyjmyer
  • Posts: 48
  • Joined: May 11, 2022
|
#95662
katehos wrote: Tue May 31, 2022 12:33 pm Hi Emily,

The main issue with answer choice (B) is that it does not address the official's actual argument. The official did not argue that raising the fines would results in another 50% (or more) decrease in parking violations, they argued that raising the fines would decrease parking violations by an indefinite amount. So, how can the flaw in the argument be something the argument never concluded in the first place? It can't!

I hope this helps! :)
-Kate
Hi Kate!

The part about connecting the answer to the conclusion definitely makes sense. That helps a lot and I think that that could be a source of a lot of my wrong answers. But, I feel like this does connect to the conclusion. The conclusion says "even fewer" and the answer choice says "at least as much as it did the first time." Doesn't that imply that the parking violations should be even fewer than the first time. Or is it wrong because the answer choice allows it to be equal to the previous number of violations, so then it would not be fewer? Still, I am not sure how this does not connect to the conclusion.

Thanks!
User avatar
 katehos
PowerScore Staff
  • PowerScore Staff
  • Posts: 184
  • Joined: Mar 31, 2022
|
#95841
Hi Emily!

I'm glad that was helpful! I definitely understand where you're coming from, but perhaps I can better illustrate what I mean through a numbered example.

Let's say that six months ago (before any fines were raised), there were about 100 parking violations on average. After the fines are raised and the garage was opened, we now only have 50 parking violations (a reduction of 50%). The Official then says raising fines again will result in even fewer parking violations. This could mean that the Official believes raising fines again will result in now only 49 violations, or 30, or 10 - we aren't sure because all of these are even fewer than 50! In order to assert what (B) says, the Official would have to be arguing that there would now be only 25 parking violations or fewer (which equates to the same 50% reduction seen the first time). However, as previously discussed, we can't really say that's what the Official is arguing.

Does that help better illustrate why (B) is incorrect? Please let me know! I'm happy to explain differently if it doesn't :)
Kate
User avatar
 christinecwt
  • Posts: 74
  • Joined: May 09, 2022
|
#97462
Hi Team - sorry that I don't understand why Answer Choice E is correct while Answer Choice C is wrong? Grateful if you can explain. Many thanks!
 Adam Tyson
PowerScore Staff
  • PowerScore Staff
  • Posts: 5191
  • Joined: Apr 14, 2011
|
#97497
Answer C is not a flaw, christinecwt, because while it's true that the author makes no mention of any financial benefits of parking fines, that isn't relevant to their conclusion. The author is not saying that we DO want fewer parking violations. They are saying IF we want fewer violations, we should again raise the fees.

The real problem with the argument is that the author assumes that it's the higher fees that caused the number of violations to go down. But there is a more obvious explanation right in front of our faces - maybe parking violations went down because there is now more legal parking available thanks to the new parking garage? Answer E is saying that the author failed to rule out that alternate cause for the decrease in violations, and "fails to rule out an alternate cause" is a classic causal flaw.
User avatar
 CristinaCP
  • Posts: 28
  • Joined: Sep 17, 2023
|
#105805
Hi!

I just want to confirm whether my reasoning for eliminating A is correct... at first I considered it because this argument is causal, and one of the ways to weaken a causal argument is to reverse the causal relationship. But then I realized that there's no way the relationship could be reversed here, because the increased fees came before the reduction in parking violations. So there's no possibility that the reduction could have caused the increased fees. Does that sound right?
User avatar
 Dana D
PowerScore Staff
  • PowerScore Staff
  • Posts: 276
  • Joined: Feb 06, 2024
|
#105837
Hey Christina,

Yes, the temporal relationship between the fines and the number of parking violations is pretty clearly established in the stimulus, so if we were looking for a flaw that would not be it. For (A) to be the correct answer, the official would have to have said that the drop in parking violations was what caused the fines to be raised, and that is not what the official says.

Get the most out of your LSAT Prep Plus subscription.

Analyze and track your performance with our Testing and Analytics Package.